(x^2-4x+3) + (3x^2-3x-5) ASAP PLS

Answers

Answer 1

Answer:

(x - 2)(4x + 1)


Related Questions

Danny is opening a savings account with an initial deposit of 45$. He saves $3 per day

Answers

The equation of the line is slope intercept form is y = 3x + 45.

What is a graph?

The set of ordered pairings (x, y) where f(x) = y makes up the graph of a function.

These pairs are Cartesian coordinates of points in two-dimensional space and so constitute a subset of this plane in the general case when f(x) are real values.

Given, Danny is opening a savings account with an initial deposit of $45 and he saves $3 per day.

Let y be the total amount he saves and x is the no. of days.

As he starts with $45 deposit it'll be added as a constant.

∴ The equation of the required line is y = 3x + 45.

To learn more about graphs here :

brainly.com/question/2288321

#SPJ9

Given f(x)=5-3x, if f(x)=-19, find x.

Answers

f(x)=5-3x
-19=5-3x
-24=-3x
24=3x
x=24/3
x=8

Bug S Bug S and Bug F is fast. Both bugs start at 0 on a number line and move in the positive direction. The bugs leave 0 at the same time and move at constant speeds. Four seconds later, F is at 12 and S is at 8. When will F and S be 100 units apart?

Answers

Answer:

Let's call the speed of Bug F v_F and the speed of Bug S v_S. Since both bugs started at 0, we can express their positions at any time t as:

Position of Bug F = 12 + v_F * t

Position of Bug S = 8 + v_S * t

To find out when F and S will be 100 units apart, we need to find the time t at which their positions differ by 100 units. In other words, we need to solve the following equation:

|12 + v_F * t - (8 + v_S * t)| = 100

We can simplify this equation by expanding the absolute value and rearranging the terms:

|4 + (v_F - v_S) * t| = 100

Now we can split this equation into two cases:

Case 1: 4 + (v_F - v_S) * t = 100

In this case, we have:

v_F - v_S > 0 (since Bug F is faster)

t = (100 - 4) / (v_F - v_S)

Case 2: 4 + (v_F - v_S) * t = -100

In this case, we have:

v_F - v_S < 0 (since Bug S is faster)

t = (-100 - 4) / (v_F - v_S)

Since we're only interested in positive values of t, we can discard the second case. Therefore, the time at which F and S will be 100 units apart is:

t = (100 - 4) / (v_F - v_S)

t = 96 / (v_F - v_S)

We don't know the values of v_F and v_S, but we can use the fact that Bug F is at 12 and Bug S is at 8, four seconds after they started. This gives us two equations:

12 = 4v_F + 0v_S

8 = 4v_S + 0v_F

Solving these equations for v_F and v_S, we get:

v_F = 3

v_S = 2

Substituting these values into the equation for t, we get:

t = 96 / (3 - 2)

t = 96

Therefore, F and S will be 100 units apart 96 seconds after they start.

Suppose you have income of $24, the price of x is $2, the price of y is $4. Your utility is given by the function U=3x^2/3y^1/3. Solve for utiltiy maximizing bundle. Suppose the government intewrvenes in this market and limits purchases of x to no more than 4 units . Are you better off? You need to demonstrate graphically or with calculations

Answers

Answer:

Step-by-step explanation:

To find the utility-maximizing bundle of goods, we need to solve for the values of x and y that maximize U while still satisfying the budget constraint. The budget constraint can be written as:

2x + 4y = 24

or

x + 2y = 12

We can use the method of Lagrange multipliers to solve for the utility-maximizing values of x and y subject to this constraint. The Lagrangian function is:

L = 3x^(2/3)y^(-1/3) + λ(x + 2y - 12)

Taking partial derivatives with respect to x, y, and λ, we get:

dL/dx = 2x^(-1/3)y^(-1/3) + λ = 0

dL/dy = -x^(2/3)y^(-4/3) + 2λ = 0

dL/dλ = x + 2y - 12 = 0

Solving these equations simultaneously, we get:

x = 6

y = 3

So the utility-maximizing bundle is 6 units of x and 3 units of y.

To see if the individual is better off with the government intervention, we can plot the budget line and the indifference curve for the utility-maximizing bundle with and without the limit on x.

Without the limit, the budget line is the same as before (x + 2y = 12), and the indifference curve for the utility-maximizing bundle passes through the point (6, 3) on the graph.

With the limit, the budget line becomes x = 4, since the individual is prohibited from purchasing more than 4 units of x. The corresponding budget line has a slope of -1/2 and intercepts the y-axis at 6.

If we draw the indifference curve for the utility-maximizing bundle of (4,4), which lies on the budget line, we can see that the individual is not better off with the government intervention. This is because the slope of the budget line under the intervention is steeper, so the individual would have to give up more y than x to afford the same amount of utility. Thus, the individual would have to move to a lower indifference curve with lower utility.

Therefore, the individual is not better off with the government intervention.

Based on the family the graph below belongs to, which equation could represent the graph?

On a coordinate plane, a curve starts at (0, 2) and curves up and to the right in quadrant 1.
y = 2 Superscript x Baseline 3
y = log (2 x) + 3
y = 2 x squared + 2
y = StartFraction 1 Over 2 x EndFraction + 2

Answers

Answer:

Second option [tex]y=\text{log}(2\text{x})+3[/tex]

Solution:

Based on the family of graphs shown in the attached file, the equation could represent the graph is [tex]y=\text{log}(2\text{x})+3[/tex]

This graph is the graph of the function [tex]y=\text{log}(\text{x})[/tex] stretched horizontally by a factor of 2 and translated 3 units upward.

Answer:

B

Step-by-step explanation:

Edge 2023

What will be the result of substituting 2 for x in both expressions below?

Answers

Substituting for x in an expression means replacing the variable x with a specific value or expression. This is often done to evaluate the expression for that particular value or to simplify the expression.

What is the substituting for x in expressions?

Substituting 2 for x in the first expression, we get:

[tex]1/2(2) + 4(2) + 6 - 1/2(2) - 2 = 1 + 8 + 6 - 1 - 2 = 12[/tex]

Substituting 2 for x in the second expression, we get:

[tex]2(2) + 2 - 1 = 5[/tex]

One expression equals 5 when substituting 2 for x, and the other equals 2 because the expressions are not equivalent.

Therefore, the first expression evaluated with x = 2 is 12, and the second expression evaluated with x = 2 is 5. Since they do not have the same value, the correct option is:

Learn more about expressions here:

https://brainly.com/question/14083225

#SPJ1

The given question is incomplete. The complete question is given below:

What will be the result of substituting 2 for x in both expressions below? One-half x + 4 x + 6 minus one-half x minus 2 Both expressions equal 5 when substituting 2 for x because the expressions are equivalent. Both expressions equal 6 when substituting 2 for x because the expressions are equivalent. One expression equals 5 when substituting 2 for x, and the other equals 2 because the expressions are not equivalent. One expression equals 6 when substituting 2 for x, and the other equals 2 because the expressions are not equivalent.

The box plots show the weights, in pounds, of the dogs in two different animal shelters.

Weights of Dogs in Shelter A
2 box plots. The number line goes from 6 to 30. For the weights of dogs in shelter A, the whiskers range from 8 to 30, and the box ranges from 17 to 28. A line divides the box at 21. For shelter B, the whiskers range from 10 to 28, and the box ranges from 16 to 20. A line divides the box at 18.
Weights of Dogs in Shelter B

Which animal shelter has the dog that weighs the least?
shelter A

Answers

Step-by-step explanation:

The minimum weight for shelter A is not provided in the given information, but we can compare the minimum weight of shelter B with shelter A's box plot.

As per the given information, the whisker of shelter A ranges from 8 to 30, which means the minimum weight in shelter A is 8 pounds. On the other hand, the whisker of shelter B ranges from 10 to 28, which means the minimum weight in shelter B is 10 pounds. Therefore, shelter A has the dog that weighs the least.

Answer:

Your answer is correct, it's shelter A.

Step-by-step explanation:

The accompanying table shows the results from a test for a certain disease. Find the probability of selecting a subject with a negative test​ result, given that the subject has the disease. What would be an unfavorable consequence of this​ error?
The individual actually had the disease

Yes
No
Positive
320
9
Negative
12
1160
Question content area bottom
Part 1
The probability is enter your response here.
​(Round to three decimal places as​ needed.)

Answers

The probability to three decimal places is 0.036 and the unfavorable consequence of this result will be that the patients who tested negative will not be treated and as such will spread the disease.

How to solve the probability

To find the probability of an event, we often use divide the total number of possible results by the total results. In this case, we are told to find the total number of persons who tested negative for the disease by the total number of positive cases. 320 + 12 persons tested positive for the disease. The sum is 332.

The total number of negative cases recorded from this sum is 12. Thus, the probability will be 12/332 = 0.036.  An unfavorable consequence of this​ error will be that the subjects who tested negative will not be treated and can thus spread the disease.

Learn more about probability here:

https://brainly.com/question/24756209

#SPJ1

Find the value of X. Round to the nearest tenth

Answers

Answer:i don't know the answer

Step-by-step explanation: i don't konw

h (x) = (3x - 4) (x + 2)^2 (x - 5)
• (2, 0)
• (-3/4, 0)
• (4/3, 0)
• (5, 0)

Answers

The zeros of the function H(x) = (3x - 4)(x + 2)^2(x - 5) are (4/3, 0), (-2, 0), and (5, 0).

Calculating the zeros of the polynomial function

To find the zeros of the function H(x), we need to find the values of x that make the function equal to zero.

H(x) = (3x - 4)(x + 2)^2(x - 5)

Setting H(x) equal to zero, we have:

(3x - 4)(x + 2)^2(x - 5) = 0

Using the zero product property, we can see that H(x) will be equal to zero when any of the factors are equal to zero.

So, the zeros of the function H(x) are:

3x - 4 = 0, which gives x = 4/3

x + 2 = 0, which gives x = -2

x - 5 = 0, which gives x = 5

Therefore, the zeros of the function H(x) are (4/3, 0), (-2, 0), and (5, 0).

Read more about polynomial at

https://brainly.com/question/7693326


#SPJ1

Find the sum. -
3/4 + 1/2 =

Answers

Answer:

Is the problem -3/4 + 1/2 or 3/4 + 1/2?

I'll just do both then.

-3/4 + 1/2 = -1/4

3/4 + 1/2 = 5/4 or 1 1/4

Step-by-step explanation:

You're welcome.

Answer:

[tex] \frac{3}{4 } + \frac{1}{2} [/tex]

take lcm of denominators i.e. 4and 2

so, the lcm of 4 and 2 is 4.

[tex] \frac{3}{4} + \frac{2}{4} [/tex]

[tex] \frac{3 + 2}{4} [/tex]

[tex] \frac{5}{4} [/tex]

Step-by-step explanation:

hope this will be helpful:)

This graph shows the solutions to the inequalities y > 3x + 14 and y < 3x + 2.
Does the system of inequalities have solutions? If so, which region contains
the solutions?
A
C

Answers

This graph in the figure shows the no solutions to the inequalities

Define inequalities

inequalities can be defined as if two real numbers or the algebraic expressions can be related by the symbols “>”, “<”, “≥”, “≤”, then the relation is called inequalities.

For example, x>3 (x should be greater than 3)

given:

system of inequalities y>3x+14 and y<3x+2

the system of inequalities have solutions only when the shaded part satisfies the given inequalities

We are given with the graph of inequalities . The given inequalities have 'and' in between.

so we look at the graph where both inequalities intersects

From the graph we can see that there is no intersection part of both shaded portions

So , we conclude that there is no solution.

to know more about real numbers, visit:

https://brainly.com/question/551408

#SPJ1

The complete question is:

This graph shows the solutions to the inequalities. does the system of inequalities y>3x+14 and y<3x+2. Does the system of inequalities have solutions? If so, which region contains the solutions?

3) Given that f(x) = 3x – 5 g(x) = 2x – 6 and h(x) = x + 4
4 2x
Find:- i) f(-3) = ii) g[f(0)] = iii) f[h(2)] =

iv) hᴏf(x) v) h-1(1) =

Answers

The Answer for the given functions are:

i) f(-3) = -14.

ii) g[f(0)] = -16.

iii) f[h(2)] = 13.

iv)  hᴏf(x) = 3x - 1.

v)  h-1(1) = -3.

What is the functiοn nοtatiοn?

Functiοn nοtatiοn is a way οf representing a functiοn using algebraic symbοls. It is a shοrthand way οf expressing a relatiοnship between twο quantities οr variables, where οne variable depends οn the οther.

i) Tο find f(-3), we substitute x = -3 in the expressiοn fοr f(x) and simplify:

f(-3) = 3(-3) - 5 = -9 - 5 = -14

Therefοre, f(-3) = -14.

ii) Tο find g[f(0)], we first evaluate f(0) and then substitute that value intο g(x):

f(0) = 3(0) - 5 = -5

g[f(0)] = g(-5) = 2(-5) - 6 = -10 - 6 = -16

Therefοre, g[f(0)] = -16.

iii) Tο find f[h(2)], we first evaluate h(2) and then substitute that value intο f(x):

h(2) = 2 + 4 = 6

f[h(2)] = f(6) = 3(6) - 5 = 18 - 5 = 13

Therefοre, f[h(2)] = 13.

iv) Tο find hᴏf(x), we substitute f(x) intο the expressiοn fοr h(x) and simplify:

hᴏf(x) = h[f(x)] = f(x) + 4 = (3x - 5) + 4 = 3x - 1

Therefοre, hᴏf(x) = 3x - 1.

v) Tο find h-1(1), we need tο sοlve fοr x in the equatiοn h(x) = 1:

h(x) = x + 4 = 1

Subtracting 4 frοm bοth sides, we get:

x = 1 - 4 = -3

Therefοre, h-1(1) = -3.

To learn more about function notation, Visit

https://brainly.com/question/27915724

#SPJ1

4. Given that cos theta= 3/10 and 3pi/2 < theta < 2pi, find the exact value of each of the following:

a) sin 2theta

b) The quadrant in which the angle theta/2 is located.

B) cos theta/2

Answers

(a) The value of sin 2θ is -3√(91)/50.

(b)  θ/2 is located in the third quadrant.

(c) The value of cos θ/2 is -√65/10.

What is the value of the sine and cosine functions?

We know that cos θ = 3/10, so we can find sin θ using the Pythagorean identity:

sin² θ + cos² θ = 1

sin²θ + (3/10)² = 1

sin²θ = 1 - (3/10)² = 91/100

sin θ = ±√(91)/10

Since 3π/2 < θ < 2π,

we know that sin θ < 0, so sin θ = -√(91)/10.

a) To find sin 2θ, we can use the double angle formula:

sin 2θ = 2 sin θ cos θ

sin 2θ = 2 (-√(91)/10) (3/10)

sin 2θ = -3√(91)/50

b) To find the quadrant in which θ/2 is located, we first need to find θ/2:

θ/2 = (3π/2 + 2π)/2 = 5π/4

5π/4 is in the third quadrant, so θ/2 is located in the third quadrant.

c) To find cos θ/2, we can use the half angle formula:

cos θ/2 = ±√((1 + cos θ)/2)

Since 3π/2 < θ < 2π, we know that cos θ < 0, so we take the negative square root:

cos θ/2 = -√((1 + 3/10)/2)

cos θ/2 = -√(13/20)

Simplifying the radical by dividing both numerator and denominator by 4:

cos θ/2 = -√(13)/2√5

Multiplying numerator and denominator by √5 to rationalize the denominator:

cos θ/2 = -√65/10

Learn more about quadrant here: https://brainly.com/question/25038683

#SPJ1

I need help with my homework

Answers

To find the length of a line segment in a circle, use the formula [tex]d = 2r[/tex] [tex]sin(t/2)[/tex] , where r is the radius of the circle and t is the angle between the radii. The length of segment DE is  [tex]5[/tex] units.

What is the formula for circle segment length?

We can use the similar triangles property to find the missing length of segment DE in the given figure. Because triangles ABD and CBE are similar, we can use a proportion to find the length of DE:

[tex]CB/BE = AB/BD[/tex]

With the given values, we get:

[tex]3/6 = 5/(5 + DE)[/tex]

When we simplify and solve for DE, we get:

[tex]3(5 + DE) = 6 * 5 \s15 + 3DE = 30[/tex]

[tex]3DE = 15 \sDE = 5[/tex]

Therefore, segment DE has a length of 5 units.

Learn more about circle here:

https://brainly.com/question/28612561

#SPJ1

a restaurant menu offers tomato, broccoli, and potato soups. customers order potato soup 50% of the time, broccoli 30% of the time, and tomato 20% of the time the chef designs a simulation to estimate how many of her next 10 customers will order broccoli soup. she labels five index cards p, three cards b, and two cards t. she shuffles the cards and randomly chooses a card from the pile. she records the letter, returns the card, and draws another. she repeats this process for a total of 10 draws. she completes this simulation five times. based on the simulations, how accurate is the chef's estimation regarding broccoli soup orders?

Answers

The simulation suggests that the chef's estimation of 30% broccoli soup orders is reasonable, but actual orders will vary from one set of customers to the next.

The chef's estimation of 30% broccoli soup orders seems reasonable based on the probabilities given, but the actual number of broccoli soup orders will likely vary from one set of 10 customers to the next. To estimate the accuracy of the chef's estimation, she conducts a simulation by shuffling cards representing the soup choices and randomly selecting one for each of the 10 customers. She repeats this process five times.

Based on the simulation results, we can see that the actual number of broccoli soup orders varies quite a bit from the expected value of 1.5. However, this is to be expected due to the random nature of the simulation.

To further estimate the accuracy of the chef's estimation, we could calculate the mean and standard deviation of the results to get a better sense of the distribution of possible outcomes.

Overall, the simulation suggests that the chef's estimation of 30% broccoli soup orders is a reasonable estimate, but the actual number of broccoli soup orders will likely vary from one set of 10 customers to the next.

To learn more about estimation please click on below link        

https://brainly.com/question/30982574

#SPJ1

Evan is going to invest in an account paying an interest rate of 5.4% compounded annually. How much would Evan need to invest, to the nearest dollar, for the value of the account to reach $1,360 in 5 years

Answers

On solving the provided question, we can say that - the value of the simple interest will be $ 881.28.

What is interest ?

Multiplying the principal by the interest rate, time, and other factors yields simple interest. Simple return equals principle times interest times hours is the marketed formula. It is easiest to compute interest using this formula. A percentage of the principle balance is how interest is most commonly computed. The interest rate on the loan is known as this percentage.

here,

we have

P = 1360;

R = 5.4 ;

T = 12

so, we get,

SI = 1360 X 5.4 X 12 /100

SI =88128/100

= 881.28

Hence, On solving the provided question, we can say that - the value of the simple interest will be $ 881.28.

To know more about simple interest visit:

https://brainly.com/question/25845758

#SPJ9

A square wrestling mat has a perimeter of (12x−32)
feet. Write an expression in simplest form that represents the side length (in feet) of the mat.

Answers

Answer:

The perimeter of a square is given by the formula P = 4s, where s is the side length. In this case, we are given that the perimeter is (12x - 32) feet, so we can set up an equation:

P = 4s

(12x - 32) = 4s

To solve for s, we can divide both sides by 4:

(12x - 32)/4 = s

Simplifying the expression on the left:

3x - 8 = s

Therefore, the expression in simplest form that represents the side length of the square wrestling mat is 3x - 8 feet.

Answer:

Step-by-step explanation:

the answer would be 3 x − 8 3x-8 3x−8 ft

16 cm
14 cm
28 cm
What is the area of the trapazoid

Answers

Area of trapezoid ((a+b):2)*h

28+14=42

42:2=21

21*16=336

Gideon took out an R150 000 loan this morning, to buy a house. The interest rate on a mortgage is 7,35%. The loan is to be repaid in equal monthly payments over 20 years. The first payment is due one month from today. How much of the second payment applies to the principal balance? (Assume that each month is equal to 1/12 of a year.)

Answers

Answer:

Step-by-step explanation:

To solve this problem, we can use the formula for calculating the fixed monthly payment on a mortgage:

P = (r * PV) / (1 - (1 + r)^(-n))

where:

P = fixed monthly payment

r = monthly interest rate (annual interest rate divided by 12)

PV = present value of the loan (loan amount)

n = total number of payments (number of years multiplied by 12)

Using the given values:

r = 0.0735 / 12 = 0.006125

PV = R150,000

n = 20 x 12 = 240

Then we can calculate the monthly payment:

P = (0.006125 * 150000) / (1 - (1 + 0.006125)^(-240)) = R1,181.91

This means that Gideon will have to pay R1,181.91 every month for 20 years to repay his loan.

To determine how much of the second payment applies to the principal balance, we need to calculate the interest and principal amounts of the first payment.

For the first payment, the interest can be calculated as:

interest1 = r * PV = 0.006125 * 150000 = R918.75

This means that the first payment consists of R918.75 in interest and the rest, R1,181.91 - R918.75 = R263.16 is principal.

To find out how much of the second payment applies to the principal balance, we need to subtract the interest and add the calculated principal amount from the first payment to the amount of the second payment:

principal2 = (P - interest1) + principal1 = (1181.91 - 918.75) + 263.16 = R525.32

Therefore, R525.32 of the second payment applies to the principal balance.

What is the GCF in simplify form

Answers

So the Greatest Common Factor of The given expressions is -63x²y, 9x³y³, and 90x³y is 9x²y.

What is expression?

In mathematics, an expression is a combination of numbers, variables, and operators (such as +, -, ×, ÷, etc.) that represents a value or a quantity. Expressions can be simple or complex, and they can involve arithmetic operations, functions, and algebraic operations. Expressions can be evaluated, simplified, or manipulated using mathematical rules and techniques. They are used in many areas of mathematics, including algebra, calculus, and geometry, as well as in other fields such as physics, engineering, and economics.

Here,

To find the greatest common factor (GCF) of the given terms, we need to factor out any common factors of the coefficients and variables.

-63x²y = (-1) × 3² × 7 × x² × y

9x³y³ = 3² × x³ × y³

90x³y = 2 × 3² × 5 × x² × y

The common factors among these terms are 3², x², and y. Therefore, the GCF of the given terms is:

GCF = 3² × x² × y

= 9x²y

To know more about expression,

https://brainly.com/question/1859113

#SPJ1

What’s 4559.886 rounded to the nearest inch

Answers

Answer:

Step-by-step explanation:

Assuming that the original measurement is in millimeters, since they are commonly used in precision applications, we know that 1 millimeter is equal to 0.03937 inches. Therefore, we can convert 4559.886 millimeters to inches by multiplying by 0.03937:

4559.886 mm * 0.03937 in/mm = 179.72467632 in (rounded to 8 decimal places)

To round this to the nearest inch, we need to look at the first decimal place after the decimal point. In this case, the digit in the first decimal place is 7, which is greater than or equal to 5. Therefore, we round up to the nearest inch, which gives us:

179.72467632 in rounded to the nearest inch is 180 in.

Therefore, 4559.886 rounded to the nearest inch is 180.

what is a prime number ​

Answers

Answer: A prime number is a positive integer greater than 1 that has no positive integer divisors other than 1 and itself. In other words, a prime number is a number that is only divisible by 1 and itself.

Step-by-step explanation:

For example, 2, 3, 5, 7, 11, 13, 17, 19, 23, 29, 31, and so on, are prime numbers because they can only be divided by 1 and themselves without any remainder.

However, 4 is not a prime number because it can be divided by 1, 2, and 4, and 6 is not a prime number because it can be divided by 1, 2, 3, and 6.

Answer:

A prime number is a number that can be multiplied by one and itself~
eg- 2,3,5,7,11
Let me know if this helps

A plane flies horizontally at an altitude of 5 km and passes directly over a tracking telescope on the ground. When the angle of elevation is /6, this angle is decreasing at a rate of /3 rad/min.

How fast is the plane traveling (in km/min) at that time? (Round your answer to two decimal places.)

Answers

Answer:

# rad u cant take 67 from it its going 35 mps

Step-by-step explanation:

Help ASAP DUE IN 30 MINUTES ​

Answers

Answer:

53 in2 is the answer for this question

Answer:

53

Step-by-step explanation:

If you divide the figure into two parts by extending the 4 in side, you get a right triangle and a rectangle.

Area of rectangle:

6*8 = 48 in²

Area of triangle:

1/2*(13 - 8)*(6 - 4) = 1/2 times 5 times 2 = 5 in²

Total area is:

48 + 5 = 53 in²

hope this helps x

Question 29 Which expression represents the distance between points J and K on the number line?​

Answers

On the number line, there are five spaces between the points J and K.

What does a number line exactly mean?

A number line is a horizontal line with consistently spaced numerical increments. How you respond to the number on the line will depend on the numbers on the line. Depending on the question that goes with it, a number can be used in a variety of ways, such as when graphing a point.

The absolute value of the difference in the coordinates between points J and K on the number line can be used to calculate the distance between them.

Points J and K in this instance have coordinates of -4 and 1, respectively. As a result, the following formula may be used to indicate the distance between locations J and K:

|1 - (-4)|

If we condense this expression, we get:

|1 + 4|

= |5|

= 5

Hence, the distance on the number line between points J and K is 5.

To know more about Number Line visit:

https://brainly.com/question/16191404

#SPJ1

a number of teenagers are playing with their calculators. one of them multiplies their ages (in whole numbers) together and finds that the product is eighteen million seven hundred and twenty seven thousand two hundred. how many teenagers are in the group

Answers

Answer:

Step-by-step explanation:

We need to find the number of teenagers in the group, given that the product of their ages is 18,727,200.

To solve this problem, we need to factorize the given number into its prime factors and then determine how many distinct factors there are.

18,727,200 can be factorized as:

18,727,200 = 2^6 × 3^2 × 5^2 × 13^2

To find the number of distinct factors, we add 1 to each exponent and then multiply them together:

(6+1) × (2+1) × (2+1) × (2+1) = 7 × 3 × 3 × 3 = 189

Therefore, there are 189 factors of 18,727,200, which means that there are 189 ways to multiply whole numbers together to get this number.

Since we want to find the number of teenagers in the group, we need to look for combinations of factors that result in whole numbers for the ages. We can start by dividing the total number of factors by 2 (since we are looking for pairs of factors) and then slowly increase the divisor until we find the smallest number that results in a whole number.

189 ÷ 2 = 94.5 (not a whole number)

189 ÷ 3 = 63 (not a whole number)

189 ÷ 4 = 47.25 (not a whole number)

189 ÷ 5 = 37.8 (not a whole number)

189 ÷ 6 = 31.5 (not a whole number)

189 ÷ 7 = 27 (a whole number)

Therefore, there are 27 pairs of factors that result in whole numbers for the ages. Each pair corresponds to a group of teenagers, and since each group has the same number of teenagers, there are 27 teenagers in the group.

PLS HURRY I AM GIVING BRAINLIEST!!!
the question is in the photo!!

Answers

Using functions,

Part A: f (3) = 6(3) + 14 = 32.

g (3) = 3 + 2 = 5

Part B: (f/g) (3) will be 6.4.

What are functions?

The core concept of calculus in mathematics is a function. The relations are certain kinds of the functions. In mathematics, a function is a rule that produces a different result for every input x. In mathematics, a function is represented by a mapping or transformation. Letters like f, g, and h are widely used to indicate these operations. The set of all potential values that can be passed into a function while it is specified is known as the domain. The entire set of values that the function's output is capable of creating is referred to as the "range." The range of potential values for a function's outputs is known as the co-domain.

Here in the question,

f (x) = 6x + 14

g (x) = x + 2

Now we have to find for the value of x,

f (3) = 6(3) + 14 = 32.

g (3) = 3 + 2 = 5

So, f (3) = 32 and g (3) = 5.

Now,

(f/g) (3) will be 32/5 = 6.4.

To know more about functions, visit:

https://brainly.com/question/12431044

#SPJ1

Which function represents the exponential graph given that (0, 1) and (4, 81/16 ) are points that lie on the graph?

f(x)=(2/3)^x

f(x)=(4)^x

f(x)=(1)^x

f(x)=(3/2)^x​

Answers

The f(x) = (3/2)ˣ is the correct function that represents the given exponential graph.

What is exponential graph?

An exponential graph is a type of graph that represents an exponential function. An exponential function is a mathematical function in which a constant base is raised to a variable exponent. The general form of an exponential function is: y = a * bˣ.

What is graph?

A graph is a visual representation of data or information that allows us to quickly and easily understand patterns, trends, and relationships. Graphs are used to present information in a way that is easy to interpret and understand.

In the given question,

The function that represents the exponential graph given that (0, 1) and (4, 81/16 ) are points that lie on the graph is:

f(x) = (3/2)ˣ

To verify that this is the correct function, we can substitute x=0 and x=4 into the function to see if the corresponding y-values match the given points on the graph.

When x=0, f(0) = (3/2)⁰ = 1, which matches the point (0, 1).

When x=4, f(4) = (3/2)⁴ = 81/16, which matches the point (4, 81/16).

Therefore, f(x) = (3/2)ˣ is the correct function that represents the given exponential graph.

To know more about exponential graph, visit:

https://brainly.com/question/9834848

#SPJ1

Show your work.
Are the expressions (4c + 8) + 3c - 2 and 4c + 2(0.5c + 1) equivalent?

Answers

First, let's simplify the expression (4c + 8) + 3c - 2:

(4c + 8) + 3c - 2

= 4c + 3c + 8 - 2 (rearrange terms)

= 7c + 6 (combine like terms)

So the first expression simplifies to 7c + 6.

Next, let's simplify the expression 4c + 2(0.5c + 1):

4c + 2(0.5c + 1)

= 4c + 2(0.5c) + 2(1) (distribute the 2)

= 4c + c + 2 (simplify the parenthesis)

= 5c + 2 (combine like terms)

So the second expression simplifies to 5c + 2.

Now we can compare the two expressions:

7c + 6 vs. 5c + 2

These expressions are not equivalent because they do not have the same coefficients or constants.

Other Questions
nonoverlapping regions that are used for planning, purchasing, and evaluating tv audiences and which are generally a group of counties in which stations located in a metropolitan or central area achieve the largest audience share are known as . cross sourcing is a strategy in which two suppliers are used for the same purchased product or service. question 1 options: true false describe some initial experminents that ouwld be needed to be conducted in order to find the rate law for the overall reaction please helppppp ill give brainlist For problems 6 and 7 set up a proportion and solve. Be quick pleaase I will give brainleiets it you tell me please please please help ill give brainlist since harley-davidson owners share a set of values that differ in certain respects from those of the overall culture, they would be classified as a(n) Quick help on b please. Company Y shows the following quarterly data relating to its widget line for the first quarter of 2015, for sales of 10,000 units:Net income $80,000Tax Rate 35%CM Ratio 45%Fixed Costs $100,000 bethany used data of the money she earned over the past 5 years to estimate the amount she would earn 10 years from now. this is an example of: group of answer choices a project being analyzed by pert has 60 activities, 13 of which are on the critical path. if the estimated time along the critical path is 214 days with a project variance of 100, what is the probability that the project will take 224 days or more to complete? group of answer choices 0.0126 near zero 0.1587 0.14 0.8413 Read each question and choose the best answer.After the Communists won the civil war, they decided to A. deliver on their promises that all political and economic power would rest with the people. B. allow the Communist party to rule as a dictatorship. C. ally with China to form a communist bloc. D. turn to full democracy after a transitional period.I need help with this question!! Answer quick for 10 points! Could someone please help me I dont understand this tom has $10,000 in nominal savings which earns 5% interest each year. if inflation were to rise to 20%, what would happen to the real value of tom's savings by the end of one year? (hint: real savings your medical patient seen today needs long term hemodialysis services. you telephone for authorization to get verbal approval. four important items to obtain are? The diagram shows a stage in the water cycle. What process is occurring at A in the diagram? Question 2 options: Water is being absorbed through roots of plants Water is being released from plants through transpiration Water is condensing on the surface of plant leaves Water is evaporating from the ground under plants Which reason best explains why the population density of Western China is so low? A. Most of the area is covered with marshes. B. Most of the area is mountainous. C. Most of the area is covered with forests. D. Most of the area is a desert. By the 46th day, there are 400 water lilies in the pond. the estimate you made:____.a. close to 46 (or exactly right) because it was estimated well.b. too low because the quick exponential growth was not accounted for. c. too high because the exponential growth was overestimated. a company that produces sporting equipment for senior citizens would like to use display ads to reach new customers. their product line gears toward people, with or without an identified interest in sports, over the age of 65. which option can best help them reach these users?